What is the solution to this system of equations?
3b + 4c = 3
8b+ 5c = 27
2b + 6c = 1

What Is The Solution To This System Of Equations?3b + 4c = 38b+ 5c = 272b + 6c = 1

Answers

Answer 1

Here ya are hope this helps! :]]

What Is The Solution To This System Of Equations?3b + 4c = 38b+ 5c = 272b + 6c = 1

Related Questions

PLEASE HELP ASAP!! WILL GIVE BRAINLIEST
A regular polygon has 12 sides. If one of its angles measures (8h − 30)°, what is the value of h?

h = 10.75
h = 18.75
h = 20.25
h = 22.5

Answers

Answer: h = 22.5

Step-by-step explanation:

Total angle area of a 12 sided polygon is 1800

1. One angle would be 1800/12= 150

8h - 30 = 150

2. Add 30 one each side to cancel out and only leave 8h.

8h = 180

3. To get rid of the 8 we divide the the other side by 8.

h = 22.5

The answer is 22.5

Answer: D 22.5 i got it right on the exam

Step-by-step explanation:

I would need to graph the line through the given point with the given slope, only help me answer question 12 please. Thank you.

Answers

The graph of the linear equation for question 12 can be seen in the image at the end of the answer.

How to graph the line?

If we have a linear equation with a slope m and a point (h, k=, the point-slope form of that linear equation is:

y = m*(x - h) + k

Here the slope is m = -7/4 and the point is (2, -6)

Then the linear equation is:

y = (-7/4)*(x - 2) - 6

To graph the linear equation we need two points on it, and we already know one, so we need to find the other.

To do so we can evaluate in some value of x.

If we use x = -2 we will get:

y = (-7/4)*(-2 - 2) - 6

y = (-7/4)*(-4) - 6

y = 7 - 6 = 1

So we also have the point (-2, 1)

Now we just need to graph the two given points and connect them with a line.

Below you can see the graph.

Learn more about linear equations:

https://brainly.com/question/1884491

#SPJ1

A rectangle has an area of 14y³ - 63y.

a. What are expressions for the length and width where one dimension is
the GCF?


pls help.

Answers

The length and breadth expressions where one dimension is the GCF are; 7y and (2y²-5).

Area of a Rectangle

Given that a rectangle is a two-dimensional shape with one of the dimensions being the GCF, as specified in the question;

Consequently, the length and breadth expressions are 7y and (2y²-5) respectively.

Additionally, the rectangle's area when y=2 is; 14(2³) - 35(2) = 52.

Learn more about how to compute the area at:

https://brainly.com/question/25292087

Abigail Henderson has $3400.00 budgeted for spending money on an upcoming trip to Country A and Country B. Country A's currency is trading at $1.30 per currency A, and Country B's currency is trading at $1.60 per currency B. She plans to spend more time in Country A, so she wants to have four times as much of currency A as currency B. Set up and solve a system of equations to model this problem, and explain what the answer means in practical terms.

Answers

If Abigail Henderson has $3400.00 budgeted for spending money on an upcoming trip .The currency needed is 200 euro and 500 pounds.

How to find the currency needed?

Formulate an equation

1.30x + 1.60y = 3400

Hence,

x =4y Equation 1

1.30x + 1.60y = 3400 Equation 2

Substitute eq1 into eq2

1.30(4y) + 1.60y = 3400

5.2y +1.60y =3400

6.8y =3400

Divide both side by 6.8y

y =3400/6.8

y = 500

Number of euro

4y where y = 500

4(500)

=2000

Therefore  the answer in practical terms is : 2000 euro of European currency and 500 pounds of Britain currency are needed.

Learn more about currency needed here: https://brainly.com/question/24373500

#SPJ1

A rectangular playground is enclosed by 440 feet of fencing. If the length of the playground is 20 feet less than 3 times the width, find the dimensions of the playground.

Answers

The dimensions of the playground are;

Width = 60 feet

Length = 160 feet

How to determine the dimensions

The formula for calculating perimeter of a rectangle is expressed as;

Perimeter = 2( l + w)

Where;

l is the length of the rectanglew is the width of the rectangle

We have the values as;

Perimeter = 440 feet

Length = 3w - 20

Width = w

Now, substitute the values into the formula, we have;

440 = 2( 3w - 20 + w)

expand the bracket

440 = 2( 4w - 20)

440 = 8w - 40

collect like terms

8w = 480

Make 'w' the subject of formula

w = 60 feet

Length = 3(60) - 20

Length = 180 - 20

Length = 160 feet

Hence, the values are 60feet and 160 feet

Learn more about rectangles here:

https://brainly.com/question/25292087

#SPJ1

Marissa has a savings account with $48 in it that earns 2% simple
interest per year.
How much money, to the nearest penny, will Marissa have in 7 years?
Give your answer in dollars.

Answers

Marissa will have $720 in 7 years when she has $48 in her saving account that earns 2% simple interest per year.

What is simple interest?Simple interest is a quick and simple approach to figuring out how much money has accrued interest. Interest is always applied to the original principal amount and is calculated at the same rate for each period of time. Any bank where we deposit money will pay us interest on that amount. Simple interest is one of many forms of interest that banks charge. Now, let's first clarify what a loan is before delving further into the idea of simple interest.S.I. = P R T,

where

P = Principal, R = Rate of Interest in% per annum, and T = Time periodThe interest rate is expressed as r/100 and is expressed as a percentage or r%.Principal: The principal is the sum that was first invested or borrowed from the bank. P stands for the principal.Rate: Rate is the interest rate at which the principal sum is granted to someone for a specific period of time; examples of rate interest are 5%, 10%, and 13%. R stands for the interest rate.Time: The amount of time that the principal is given to someone. T represents time.Amount: When a borrower repays a bank loan, the term "Amount" refers to the sum of the principal borrowed plus the interest paid.Amount = Principal + Simple Interest or A = P(1 + RT)

Calculation

From, A = P(1 + RT)

where, p = $48

R = 2%

T= 7 years

A = 48(1 + 2*7)

A = $720

Hence, Marissa will have $720 in her bank account after 7 years.

To learn more about simple interests, refer to https://brainly.com/question/20690803

#SPJ1

right answers only please

Answers

Given graph represent the graph of  g(x) = [tex]-x^{2} (x -2)[/tex]

Given,

In the question;

The equation is:

g(x) = - [tex]x^{2} (x -2)[/tex] or h(x) = [tex]x^{2} (x -2)[/tex]

To find the is this the graph of g(x) or h(x).

Graph is given in the figure.

Now, According to the question;

g(x) = [tex]-x^{2} (x -2)[/tex]        and      h(x) = [tex]x^{2} (x -2)[/tex]

g(x) = 0                                    h(x) = 0

[tex]-x^{2} (x -2) = 0[/tex]                         [tex]x^{2} (x -2)[/tex] = 0

x = 0, 2                                    x = 0, 2

If we put x = 1 then,

g(1) = [tex]-(-1)^2(1-2)[/tex]     and   h(1) = [tex](1)^2(1-2)[/tex]

g(1)  =  -1 (-1)                          h(1) = 1(-1)

g(1)   = 1                                 h(1) = -1

But as shown in graph, graph take positive value at x = 1

∴Given graph is the graph of g(x) = [tex]-x^{2} (x -2)[/tex]

Hence, Given graph represent the graph of  g(x) = [tex]-x^{2} (x -2)[/tex]

Learn more about Graph at:

https://brainly.com/question/10712002

#SPJ1

Find the solution set of 12x-31 + 1 = 6.
x = 1 or x=-1
x = 4 or x = -1
x = 5 orx = -2
x=4 or x = -4

Answers

x = 4 or x = -1 :)
hope this helps!!

In the middle of a park, there is a fenced off area in the shape of a square with an area of 225 yards. What is the side length of the square? What is the perimeter of the fenced off area?

what is a peremiter ​

Answers

Perimeter meaning: the continuous line forming the boundary of a closed geometric figure.

side length: 15

perimeter: 368

1/3[6y+3]-24=3[1/3y-6]
y-3=y-7
x+2x=1/3x-8
y-9=y-2
x-5=6x-5



step by step explanation please

Answers

Answer:

Step 1: Simplify both sides of the equation.

1/3(6y+3)−24 = 3(1/3y−6)

1/3(6y) + 1/3(3) -24 = 3(1/3y) + 3(-6)

2y +1 −24 = y−18

2y −23 = y −18

Subtract y from both sides.

2y−23−y=y−18−y

y−23=−18

Step 3: Add 23 to both sides.

y−23+23 = −18+23

y = 5

Step-by-step explanation:

What is the slope of the line?

Answers

Answer:

2

Step-by-step explanation:

y    4-2=2

x     2-1=1    

2/1=1

hope it helps.

Examine this set of Pythagorean triples. Look for a pattern that is true for each triple regarding the difference between the three values that make up the triple.

Describe this pattern. Then see if you can think of another Pythagorean triple that doesn’t follow the pattern you just described and that can’t be generated using the identity . Explain your findings.

x-value Pythagorean Triple
3
4
5
6

Answers

The pattern is that two larger numbers differ by 2 and the smaller is the root of twice the sum of the larger two.

How to explain the pattern?

The given formula gives rise to the above observations. it tells you

a = 2x

b = x^2 -1

c = x^2 +1

This is a special case of Euclid's formula ...

a = 2mn

b = m^2 -n^2

c = m^2 +n^2

for n=1.

Since m and n can be any integer values, there are certainly many other. Pythagorean triples that do not match the formula given in the problem. Using n=2, for example, the triples are:

a = 4x

b = x^2 -4

c = x^2 +4

Some triples from this set of formulas are (5, 12, 13), (20, 21, 29), (28, 45, 53), (36, 77, 85).

It should bm be noted that triples from this set of formulas will not match directly those from the formulas in the problem statement, but triples from both lists may reduce to the same primitive triple.

Learn more about Pythagoras on:

https://brainly.com/question/11279692

#SPJ1

Complete question

Examine this set of Pythagorean triples from part C. Look for a pattern that is true for each triple regarding the difference between the three values that make up the triple.

Describe this pattern. Then see if you can think of another Pythagorean triple that doesn’t follow the pattern you just described and that can’t be generated using the identity (x2 − 1)2 + (2x)2 = (x2 + 1)2. Explain your findings.

x-value Pythagorean Triple

3 (6,8,10)

4 (8,15,17)

5 (10,24,26)

6 (12,35,37)

Which equation below represents the Exponent Property of Logs?

Answers

The only equation given that represents the Exponent Property of Logs is; Option A: log(A^r) = r*log(A)

How to find the exponent property of logarithm?

The  exponent property of logarithm says that the log of a power is the exponent times the logarithm of the base of the power. For example;

logₐ(MN) = logₐM + logₐN

logₐ(M/N) = logₐM - logₐN

logₐ(3²) = 2logₐ3

The first example above shows the product rule of logarithms

The second example shows the quotient rule of logarithms

The third rule shows the power rule

Now, looking at the options, the only one that shows the correct pattern of exponent property of logarithm is;

Option A: log(A^r) = r*log(A)

Read more about exponent property of logarithm at; https://brainly.com/question/28596566

#SPJ1

From their locations in the diagram, what are two possible values for n and m? The letters n and m represent two unknown numbers.

Answers

The possible values for n and m are:

m = √5 and n = 7/5

In this question, we have been given the locations of numbers n and m.

From their locations in the diagram, we need to find the possible values for n and m.

From the location of n we can say that the value of n is a rational number but not an integer.

We know that rational numbers are of the form p/q where p, q are integers with q ≠ 0

And the value of m is an irrational number.

So, the possible value of n am m:

m = √5 and n = 7/5

Therefore, the possible values for n and m are:

m = √5 and n = 7/5

Learn more about rational number here:

https://brainly.com/question/17450097

#SPJ1

The following set of four ordered pairs below defines the vertices, in counterclockwise order, of a quadrilateral (four-sided figure). {(0,−2),(6,0),(4,5),(−2,3)} Step 1 of 3 : Find the slopes of the indicated sides of the quadrilateral. Simplify your answer.

Answers

The slope of the side connecting (0,−2) and (-2, 3) is: -5/2.

The slope of the side connecting (6, 0) and (4, 5) is: -5/2.

How to Find the Slope Between Two Points?

The slope (m) of a side connecting two points with given coordinates can be calculated using the slope formula shown below:

Slope (m) = change in y / change in x = [tex]\frac{y_2 - y_1}{x_2 - x_1}[/tex].

Slope of the side connecting (0,−2) and (-2, 3):

Slope (m) = change in y / change in x = (3 - (-2)) / (-2 - 0)

m = 5/-2

Slope (m) = -5/2.

Slope of the side connecting (6, 0) and (4, 5):

Slope (m) = change in y / change in x = (5 - 0) / (4 - 6)

m = 5/-2

Slope (m) = -5/2.

Therefore, the slope of both sides are the same, which is: -5/2.

Learn more about slope on:

https://brainly.com/question/16949303

#SPJ1

Two photos are similar the ratio of the corresponding side length is 3:4 what is the ratio of their areas

Answers

The ratio of the areas of these two photos is equal to 9:16.

What is a ratio?

In Mathematics, a ratio simply refers to a mathematical expression that is typically used for denoting the proportion of two (2) or more quantities with respect to one another and the total quantities.

This ultimately implies that, a ratio is generally expressed as a quotient of two numerical quantities or variables.

Generally speaking, the corresponding ratio of the areas of two geometric figures (shapes) is equal to the square of the ratio of their corresponding side length:

Area of photos = (a:b)²

Substituting the given parameters into the formula, we have;

Area of photos = (3:4)²

Area of photos = 9:16

Read more on ratio here: brainly.com/question/28083504

#SPJ1

I am running out of points please help I have 13

Answers

Answer:

(A∩B)={1,15}

Step-by-step explanation:

9(9-10r)+(-8-2r)


I don't know how much to do this I need help

Answers

Answer: 21

Step-by-step explanation:

To simplify this, we would use Order of Operations (BEDMAS)

Brackets ()Exponents [tex]x^{2}[/tex] Division MultiplicationAdditionSubtraction

So, to simplify 3 + (3-8)^2 -7 we will follow the order of Operations

[tex]3 + (3-8)^{2} - 7\\3 + (-5)^{2} - 7\\3 + 25 - 7 \\28 - 7\\21[/tex]

So the answer is 21

Complete a trend analysis for sales (round to nearest whole percent and use 2010 as the base year). 2013 2012 2011 2010 Sales $ 620,000 $ 580,000 $ 450,000 $ 600,000 (D) (C) (B) (A)

Answers

The completion of the trend analysis for sales with 2010 as the base year is as follows:

Trend Analysis of Sales:

                             2013           2012           2011           2010

Sales            $ 620,000   $ 580,000   $ 450,000   $ 600,000

Trend Analysis      103%             97%              75%             100%

What is trend analysis?

Trend analysis is a technical analysis that uses historical data over some periods to predict and monitor future developments in a particular topic.

To undertake a trend analysis for sales, a base year is chosen and the sales account for the base year is set at 100% and used to check how sales have changed during the review periods.

The other years' results are compared to the base year's by dividing each year's result by the base year's.

Percentages are used to depict trend analysis so that the resulting increases and decreases can be easily compared to the base year.

Sales Trend Analysis:

2013 (D) = 103% ($620,000/$600,000 x 100)

2012 (C) 97% ($580,000/$600,000 x 100)

2011 (B) 75% ($450,000/$600,000 x 100)

2010 (A) = 100% ($600,000/$600,000 x 100)

Learn more about trend analysis at https://brainly.com/question/13760796

#SPJ1

Help please willing to give brainliest badge an points

Answers

The slope intercept form equation of the straight line is y = -1/2x + 6

Slope intercept form

Slope intercept form of the equation used to calculate the equation of a straight line by using the slope and intercept value.

And the general form of slope intercept equation is

y = mx + c

where

m refers the slope

c refers the y - intercept

Given,

Here we have the graph of straight line.

Through the given graph we have to identify the equation of slope intercept form.

In order to find the slope intercept form equation we need two values they are slope and the y intercept.

So, to find these two values, we need two points,

So we have selected the points from the given graph. They are (2,5) and (4,4).

Now we have to use the slope formula to find the value of the slope,

m = (4 - 5) / (4 - 2)

m = -1/2

Now, we have to use the slope and one of the point (4,4), to find the value of y intercept,

4 = -1/2 (4) + c

4 = -2 + c

c = 6

Now, we have identified the values of slope and the intercept.

Then the slope intercept form equation of these straight line is written as,

y = -1/2x + 6

To know more about Slope intercept form here.

https://brainly.com/question/9682526

#SPJ1

A=P(1+r/100)^n express r in terms of A,P, and n​

Answers

Answer:

[tex] \huge{ \boxed{r =100( \sqrt[n]{ \frac{A}{P} } - 1)}}[/tex]

Step-by-step explanation:

[tex]A = P(1 + \frac{r}{100} )^{n} \\ [/tex]

First of all in order to make r the subject we have to first divide both sides of the equation by P

That's

[tex] \frac{A}{P} = \frac{P( {1 + \frac{r}{100} })^{n} }{P} \\[/tex]

We'll finally get

[tex] \frac{A}{P} = ( {1 + \frac{r}{100}) }^{n} \\ [/tex]

Next we have to remove the power n and to do that we find the square root of 'n' of both sides as

We have

[tex] \sqrt[n]{ \frac{A}{P} } = \sqrt[n]{( {1 + \frac{r}{100} })^{n} } \\ \\ \\ \sqrt[n]{ \frac{A}{P} } = 1 + \frac{r}{100} [/tex]

Next we subtract 1 from both sides to isolate r/100

We have

[tex]\sqrt[n]{ \frac{A}{P} } - 1 = 1 - 1 + \frac{r}{100} \\ \sqrt[n]{ \frac{A}{P} } - 1 = \frac{r}{100} [/tex]

Finally to isolate r , we multiply both sides by 100

[tex] \frac{r}{100} \times 100 =100 \times (\sqrt[n]{ \frac{A}{P} } - 1)[/tex]

We have the final answer as

[tex]r =100( \sqrt[n]{ \frac{A}{P} } - 1) \\ [/tex]

Hope this helps you

2.1 & 2.2 Assessment: Vertex & Standard Form of Quadratic Functions



What is the equation, written in vertex form, of a parabola with a vertex of (1, -9) that
passes through the point (2, -7)?



The equation of the parabola in vertex form is _______________

Answers

Answer:

y = 2(x - 1)^2 - 9

Step-by-step explanation:

The vertex form of a parabola is y = a(x - h)^2 + k

(h, k) is the vertex, aka (1, -9)

(x, y) is the point that is passed through aka (2, -7)

Using this, we can substitute these values into the equation.

First, let's work with the vertex

We know that h = 1 and k = -9:  y = a(x - 1)^2 - 9

Now, let's substitute in the point that was given: -7 = a(2 - 1)^2 - 9

Now, we can find the a-value by simplifying: -7 = a - 9   -->  a = 2

Finally, we can make the equation by taking our equation that we made with the vertex but just putting the a-value in since we know it.

y = 2(x - 1)^2 - 9 is the equation of the parabola in vertex form

I hope this helps! Let me know if it is wrong


In a football game, Team A defeated Team B by a score of 31 to 14. The total points scored came from a combination of touchdowns, extra-point kicks, and field goals, worth 6, 1, and 3
points, respectively. The numbers of touchdowns and extra-point kicks were equal. There were six times as many touchdowns as field goals. Find the number of touchdowns, extra-point
kicks, and field goals scored.

Answers

The number of touchdowns is 4.

The number of extra-point kicks is 4.

and the number of field goals is 2.

What are linear equations in three variables?

If a, b, c, and r are real numbers (and if a, b, and c are not all equal to 0) then ax + by + cz = r is called a linear equation in three variables.

How to solve the system of linear equations in three variables?

Select two pairs of equations from the system. Eliminate the same variable from each pair using the Addition/Subtraction method. then, solve the system of the two newly obtained equations using the Addition/Subtraction method.

Let the number of touchdowns be "x", the number of extra-point kicks be "y", and the number of field goals is "z".

Total points scored in game = 20+14 =34

Now,

According to the given question,

                                6x + y + 3z = 34 -----------(I)

                                x = y ----------(II)

                                x = 2z

                                z = [tex]\frac{x}{2}[/tex] -----------(III)

Putting the values of "y" and "z" in equation (I), we get

                              6x + x + 3×[tex]\frac{x}{2}[/tex] = 34

                               [tex]\frac{17x}{2}[/tex] = 34

                               x = 4

Now, putting the value of "x" in equations (II) and (III), we get

                               y = 4.

                               z = 2.

Hence, The number of touchdowns is 4. The number of extra-point kicks is 4, and the number of field goals is 2.

To learn more about the system of linear equations, visit:

https://brainly.com/question/26150814

#SPJ1

A ship sails at a speed of 36 nautical miles per hour due east. An hour later it changes its direction to N 32° E and continues at the same speed and direction for a half hour. At this point, what are the ship's distance and direction of bearing from the starting point? Round your answers to the nearest hundredth:
The ship is____________ nautical miles from the starting point at a directional bearing of N _______ ° E.

Answers

The distance and direction of the ship from the starting point is found using vectors as follows;

The ship is 48.03 nautical miles from the starting point at a directional bearing of N 18.53° E

What are vectors?

A vector is a quantity that has a property of magnitude or size and direction.

The ship's speed = 36 nautical miles per hour

Direction of the ship = Due east

The duration the ship continues in the direction = 1 hour

The new direction of the ship = N32°E

The duration in which the ship continues in the new direction = Half an hour

The vector of the distances traveled by the ship are;

[tex]\overrightarrow{d_1}[/tex] = 36·i

[tex]\overrightarrow{d_2}[/tex] = 0.5 × 36×sin(32°)·i + 0.5 × 36×cos(32°)·j = 18×sin(32°)·i + 18×cos(32°)·j

The sum of the distances traveled by the ship is therefore;

36·i + 18×sin(32°)·i + 18×cos(32°)·j = (36 + 18×sin(32°))·i + 18×cos(32°)·j

(36 + 18×sin(32°))·i + 18×cos(32°)·j = 45.54·i + 15.26·j

The distance of the ship from the starting point, d, is therefore;

d ≈ √(45.54² + 15.26²) ≈ 48.03

The distance of the ship from the starting point is approximately 48.03 miles.

The direction from the North of the ship from its starting point, θ, is therefore;

[tex]\theta = arctan\left(\dfrac{15.26}{45.54} \right) \approx 18.53^{\circ}[/tex]

Then direction of the ship is N 18.53° E

Learn more about vectors here:

https://brainly.com/question/14896118

#SPJ1

foot of a 10m long ladder leaning against a vertical well is 6m away from the base of the wall. Find the height of the point on the wall where the top of the ladder reaches.

Answers

For 10m long ladder whose foot is leaning against a vertical wall and from the base of the wall it is 6m away then height of the wall at a point ladder reaches is equal to 8m.

As given in the question,

Wall , ladder and base forms a right angled triangle

Length of the ladder leaning against the wall is hypotenuse = 10m

Distance between the ladder and base of the wall is base= 6m

Let us consider 'h' be the height of the wall is altitude.

Wall , ladder and base forms a right angled triangle

Using Pythagoras theorem we have,

(Hypotenuse)² = (Base)² + (altitude)²

⇒10² = 6² + h²

⇒100 =36 +h²

⇒h² =100 -36

⇒h² =64

⇒h = 8m

Therefore, for 10m long ladder whose foot is leaning against a vertical wall and from the base of the wall it is 6m away then height of the wall at a point ladder reaches is equal to 8m.

Learn more about height here

brainly.com/question/10726356

#SPJ1

What is 1 5/9+2 7/9 in simplest form

Answers

Answer:

13/3

Step-by-step explanation:

3 [tex]\frac{12}{9}[/tex]

[tex]\frac{12}{9}[/tex]= 1 [tex]\frac{1}{3}[/tex]

3+1 1/3

4 [tex]\frac{1}{3}[/tex]

13/3

Answer:

Step-by-step explanation:

step one just add 5/9 +7/9 since they have the same denominator then that will equal 4/3

2nd step add the 1 and 2 together since they are both whole numbers. which equals 3

3rd step 4/3 + 3/1 which equals 13/3 maybe or if the answers wants a mixed number than the answer might be 4 1/3!

I'm not quite sure but try and see if its correct! :)

Use the sample data and confidence level given below to complete parts​ (a) through​ (d). A research institute poll asked respondents if they felt vulnerable to identity theft. In the​ poll, n=990 and x=581 who said​ "yes." Use a 95% confidence level.

Answers

The 95% confidence interval for the proportion of respondents who say yes is of:

(0.5562, 0.6176)

What is a confidence interval of proportions?

A confidence interval of proportions has the bounds given by the rule presented as follows:

[tex]\pi \pm z\sqrt{\frac{\pi(1-\pi)}{n}}[/tex]

In which the variables used to calculated these bounds are listed as follows:

[tex]\pi[/tex] is the sample proportion, which is also the estimate of the parameter.z is the critical value.n is the sample size.

The confidence level is of 95%, hence the critical value z is the value of Z that has a p-value of [tex]\frac{1+0.95}{2} = 0.975[/tex], so the critical value is z = 1.96.

The remaining parameters, which are the sample size and the estimate, are given as follows:

[tex]n = 990, \pi = \frac{581}{990} = 0.5869[/tex]

The lower bound of the interval is of:

[tex]\pi - z\sqrt{\frac{\pi(1-\pi)}{n}} = 0.5869 - 1.96\sqrt{\frac{0.5869(0.4131)}{990}} = 0.5562[/tex]

The upper bound of the interval is of:

[tex]\pi + z\sqrt{\frac{\pi(1-\pi)}{n}} = 0.5869 + 1.96\sqrt{\frac{0.5869(0.4131)}{990}} = 0.6176[/tex]

More can be learned about the z-distribution at https://brainly.com/question/25890103

#SPJ1

Find the values of x and y.​

Answers

Answer:

x = 20 degrees ; y = 70 degrees

Step-by-step explanation:

angle supplementary to 40 = 180 - 40 = 140 degrees


in isosceles triangle has the angles of the base with the same measure


140 + (2x) = 180

2x = 180 - 140

2x = 40

2/2 x = 40/2

x = 20 degrees


90 - 20 = 70 degrees


70 + y + 40 = 180

110 + y = 180

y = 180 - 110

y = 70 degrees


Answer:

Step-by-step explanation:

What is the geometric mean of 5 and 11? * ​

Answers

For what my understanding is the geometric mean of 5 and 11 is 7.4162, since
= ( 5 x 11 ) ^0.5
= ( 55 )0.5
= 7.4162

Productive Investments Limited provided the
returns for Laska stocks for 2017 to 2021. Use the
information supplied to answer the questions below.
Year
2017
2018
2019
2020
2021
Laska return
-2.2%
-3.4%
12.5%
-0.8%
8.6%
a) Calculate the range of the market returns. (2
Marks)
b) Calculate the mean market return for 2022 (2
Marks).
c) Calculate the variance of the market returns (6
Marks).
d) Calculate the standard deviation of the market
returns (2 Marks).
e) Comment on your results (4 Marks).

Answers

The range of market return is 15.9% and the mean of market return is 2.94%

Productive Investments Limited provided the returns for Laska stocks for 2017 to 2021.

Year         Laska return

2017         -2.2%

2018          -3.4%

2019         12.5%

2020         -0.8%

2021          8.6%

The range of market return = Highest return - lowest return

= 12.5 - (-3.4)

= 15.9%

mean market return = (-2.2 +(-3.4) + 12.5 +(-0.8) + 8.6)/5

mean market return = 2.94%

Therefore, the range of market return is 15.9% and the mean of market return is 2.94%

To learn more about data interpretation refer here

https://brainly.com/question/19243813

#SPJ9

Other Questions
japan's population decline is directly attributed to a. the legacy of the atomic bombs dropped on hiroshima and nagasaki by the united states. b. the 201l earthquake, tsunami and consequent nuclear explosion. c. a demographic shift to smaller families and more women in the workforce. d. a legacy of inadequate pollution controls. e. impoverished urban populations suffering from lack of sanitation and health care facilities. you want to increase the relevance of a search ad so it's more meaningful to potential customers and provides value-added information to their searches. what two actions might improve the relevance of your ad? (choose two.) select 2 correct responses selecting only relevant languages in campaign setup. posting transparency statements on the website. selecting distinct geographic areas. changing the ad's call-to-action statement. rewriting the landing page for clarity. which of the following is not a reason why firms use budgets? a. operating leverage. b. performance evaluation. c. coordination. d. planning. HELPPPP I'm so confused!!! 60 POINTSSSS!!! Is anyone here a car enthusiast? At a constant temperature, what is the new volume of a 5.0-L sample of oxygen gasif the pressureis changed from 0.50 atm to 0.25 atm? after a very short time, what is the difference between the pressure-difference across the component on the left and the pressure-difference across the component on the right? Use this model to describe unobservable relationships If the narrator of a story is a rattlesnake, what would the word "gliding" probably mean in the story? the nurse is caring for a patient in hospice. the nurse notes that the patient is getting adequate care, but the spouse is not sleeping well. the nurse also assesses the need for better family nutrition and meals assistance. the nurse discusses these needs with the patient and family and develops a plan of care with them using community resources. which approach is the nurse using? when you receive an offer on a listing for less than asking price, which of these would be the least effective course of action to take? You bought a stock one year ago for per share and sold it today for per share. It paid a per share dividend today. What was your realized return?. HEEEEEEEEEELPPPP!!!!!! WILL MARK AS BRAINLIEST! 100 Points which organizational structure is characterized by low flexibility, low adaptability, and low problem recognition? What characteristics made the south different from the rest of the united states during the early nineteenth century?. During 75 days, rain fell on 3/5 of the day. On how many days did rainfall? blaylock plans to sell 94,000 units of product no. 794 in may, and each of these units requires three units of raw material. pertinent data follow. product no. 794 raw material actual may 1 inventory 11,900 units 29,900 units desired may 31 inventory 17,900 units 20,900 units on the basis of the information presented, how many units of raw material should blaylock purchase for use in may production? what do you add to 5 8/9 to make 7 Complete the table: (Enter your answers as a whole dollar amount.) Units Unit Cost Dollar Cost Beg. Inventory Jan. 1 10 $ 7.00 A Apr. 11 30 $ 12.00 B May 17 40 $ 13.00 C Dec. 5 20 $ 15.00 D It can be the plants you want to describe how it is.